Multiple equality conditions

Prove that if a,b,ca, b, c are non-negative real numbers such that a+b+c=3 a + b + c = 3 , then we have

ab2+bc2+ca2+abc4. ab^2 + bc^2 + ca^2 + abc \leq 4 .

#Algebra

Note by Calvin Lin
6 years, 1 month ago

No vote yet
1 vote

  Easy Math Editor

This discussion board is a place to discuss our Daily Challenges and the math and science related to those challenges. Explanations are more than just a solution — they should explain the steps and thinking strategies that you used to obtain the solution. Comments should further the discussion of math and science.

When posting on Brilliant:

  • Use the emojis to react to an explanation, whether you're congratulating a job well done , or just really confused .
  • Ask specific questions about the challenge or the steps in somebody's explanation. Well-posed questions can add a lot to the discussion, but posting "I don't understand!" doesn't help anyone.
  • Try to contribute something new to the discussion, whether it is an extension, generalization or other idea related to the challenge.
  • Stay on topic — we're all here to learn more about math and science, not to hear about your favorite get-rich-quick scheme or current world events.

MarkdownAppears as
*italics* or _italics_ italics
**bold** or __bold__ bold

- bulleted
- list

  • bulleted
  • list

1. numbered
2. list

  1. numbered
  2. list
Note: you must add a full line of space before and after lists for them to show up correctly
paragraph 1

paragraph 2

paragraph 1

paragraph 2

[example link](https://brilliant.org)example link
> This is a quote
This is a quote
    # I indented these lines
    # 4 spaces, and now they show
    # up as a code block.

    print "hello world"
# I indented these lines
# 4 spaces, and now they show
# up as a code block.

print "hello world"
MathAppears as
Remember to wrap math in \( ... \) or \[ ... \] to ensure proper formatting.
2 \times 3 2×3 2 \times 3
2^{34} 234 2^{34}
a_{i-1} ai1 a_{i-1}
\frac{2}{3} 23 \frac{2}{3}
\sqrt{2} 2 \sqrt{2}
\sum_{i=1}^3 i=13 \sum_{i=1}^3
\sin \theta sinθ \sin \theta
\boxed{123} 123 \boxed{123}

Comments

This is quite a famous inequality, and there are several approaches that could be taken. Here is the simplest proof that I know of.

Define f(a,b,c)=ab2+bc2+ca2+abc f( a, b, c) = ab^2 + bc^2 + ca^2 + abc . WLOG, we may assume that a a is the median of the 3, IE we either have bac b \leq a \leq c or bac b \geq a \geq c .

Step 1: We will show that f(a,b,c)f(a,b+c,0) f(a,b,c) \leq f ( a, b+c, 0 ) . This approach is known as Smoothing.

This follows by expanding both sides, and we want to compare ab2+bc2+ca2+abca(b+c)2 ab^2 + bc^2 + ca^2 + abc \leq a(b+c)^2 , which simplifies to

bc2+ca2abc+ac2 bc^2 + ca^2 \leq abc + ac^2

This is equivalent to

c(ab)(ac)0 c ( a - b ) ( a - c) \leq 0

From the assumption that aa is the middle number, we know that ab,ac a-b, a-c will have different signs (or be 0). Since c c is non-negative, hence the entire product will be 0 \leq 0 .

This is essentially how we approach @Krishna Sharma 's Case 1.

Step 2: We show that subject to a+b=3 a + b = 3 , we have f(a,b,0)4=f(1,2,0) f(a, b, 0) \leq 4 = f ( 1, 2, 0) .

By AM-GM, we get

f(a,b,0)=ab2=4×a×b2×b24(a+b2+b23)3=4. f(a, b, 0) = ab^2 = 4 \times a \times \frac{b}{2} \times \frac{b}{2} \leq 4 \left( \frac{a + \frac{b}{2} + \frac{b}{2} } { 3} \right) ^3 = 4 .

Hence, the result follows.


However, I do not know of an easy way to motivate the approach, and in particular step 1. Any thoughts or comments?

Calvin Lin Staff - 6 years, 1 month ago

Log in to reply

Minor typo in the last step where you use AM-GM.

4×a×(b2)24(a+b2+b23)3=44\times a\times \left(\frac{b}{2}\right)^2\leq 4\left(\frac{a+\frac{b}{2}+\frac{b}{2}}{\color{#D61F06}{3}}\right)^3=4

Prasun Biswas - 6 years, 1 month ago

Log in to reply

Thanks fixed.

Calvin Lin Staff - 6 years, 1 month ago

Sir, What is meant by IE ?

Priyanshu Mishra - 5 years, 7 months ago

Log in to reply

It's just another way to write i.e. which means "that is".

Prasun Biswas - 5 years, 7 months ago

This is just a comment, not a solution.

I remember seeing a stronger version of this inequality somewhere recently (probably on Math SE) which stated the following:

If a,b,ca,b,c are non-negative reals, then the following inequality holds:

  ab2+bc2+ca2+abc427(a+b+c)3~\\~ab^2+bc^2+ca^2+abc\leq \frac{4}{27}(a+b+c)^3

Our required inequality trivially follows from the stated inequality. However, proving the said stronger inequality seems to be harder than I expected. Let me see if I can think of a proof to that. For the time being, others are welcome to post their proof (if any) for the stated stronger inequality.

Prasun Biswas - 6 years, 1 month ago

Log in to reply

Note that the inequalities are equivalent. What happened was that we normalized the inequality, meaning that we made all of the terms have the same polynomial degree. To do so, we multiplied, where necessary, by a+b+c=3 a + b + c = 3 .

This is a standard approach, and one that I would often (though not always) recommend to use if the terms have different degrees.


The weaker version that you saw, was most likely

ab2+bc2+ca24ab2+bc2+ca2427(a+b+c)3. ab^2 + bc^2 + ca^2 \leq 4 \Leftrightarrow ab^2 + bc^2 + ca^2 \leq \frac{4}{27} ( a + b + c ) ^ 3 .

Calvin Lin Staff - 6 years, 1 month ago

Is some info missing? Because substituting a=b=c=43a=b=c = \frac{4}{3} yields a value around 9.5

Krishna Sharma - 6 years, 1 month ago

Log in to reply

Ooops, the condition should have been a+b+c=3 a+b+ c = 3 .

Calvin Lin Staff - 6 years, 1 month ago

Let f(a,b,c)=ab2+bc2+ca2+abc+k(a+b+c3)f(a,b,c)=ab^2+bc^2+ca^2+abc + k(a+b+c-3) (say this as equation (1)(1)

Solving these four equations fa=0\frac{\partial f}{\partial a} = 0 fb=0\frac{\partial f}{\partial b} = 0 fc=0\frac{\partial f}{\partial c} = 0 a+b+c=3a+b+c=3

we get a=b=c=1,k=4a=b=c=1 , k=-4

substituting this back in equation 11 , we get fmax(a,b,c)=1+1+1+14(0)=4f_{max}(a,b,c)=1+1+1+1-4(0) = 4

Q.E.D\textbf{Q.E.D}

@Calvin Lin sir

Aman Rajput - 5 years, 9 months ago

Log in to reply

This demonstrates the point of this problem.

You did not perform the Lagrangian properly. At the IMO, this solution will be scored 0/7.
1. You did not state the equations.
2. You did not state how to solve the equations.
3. You missed out the equality case of (1,2,0) (1, 2, 0) .

Calvin Lin Staff - 5 years, 9 months ago

Log in to reply

I didn't miss the case 1,2,0 doesnt satisfy the third equation ,i.e. , df/dc = 0

Aman Rajput - 5 years, 9 months ago

Log in to reply

@Aman Rajput You missed it because you didn't take care of the boundary condition. IE You didn't perform the Lagrangian properly.

E.g. What is the maximum of f(x)=x2 f(x) = x^2 on the interval [2,2] [-2, 2] ? Do you say that " f=0x=0f' = 0 \Rightarrow x = 0 hence the maximum is f(0)=0 f(0) = 0 ? No, we still have to check the boundary points, where they need not satisfy f=0 f' = 0 in order to be a maximum on the restricted domain.

Calvin Lin Staff - 5 years, 9 months ago

Log in to reply

@Calvin Lin I know what you are trying to say . But what i know is that the min / max will be obtained using lagrangian even if we missed out boundary condition or other ordered pairs of equality

Aman Rajput - 5 years, 9 months ago

Log in to reply

@Aman Rajput No it will not.

For example, if you ignore the boundary condition when calculating the max of f(x)=x2 f( x) = x^2 on the closed interval [1,1] [1, -1] , then you would not get any answer.

Calvin Lin Staff - 5 years, 9 months ago

Log in to reply

@Calvin Lin Okay ,, ya agree ! in this question we have to check at boundary.

ok i will keep in mind always to check at boundary

Aman Rajput - 5 years, 9 months ago

This problem becomes quite a standard exercise when you use Lagrange multipliers. I think that the challenge is to prove the given inequality without using that which would be the reason for the problem being tagged under Algebra.

Prasun Biswas - 5 years, 9 months ago

Log in to reply

i dont think so ... :)

Aman Rajput - 5 years, 9 months ago

Log in to reply

@Aman Rajput What Prasun said is actually true. Lagrange Multiplier makes this problem too easy. It's like you're using a chainsaw to cut a carrot.

On the other hand, you need to show that the extremal point you've found is a global maximum point as opposed to an inflection point or a global minimum point.

Pi Han Goh - 5 years, 9 months ago

Why not assume a=b= c=1 and do it

shailesh hegde - 6 years, 1 month ago
×

Problem Loading...

Note Loading...

Set Loading...